« first day (39 days earlier)      last day (535 days later) » 
00:00 - 16:0016:00 - 00:00

12:04 AM
0
Q: show that this function is differentiable and not increasing

user297759Abbott: understanding analysis I'm going through exercises in my book but I can't figure this one out.

0
Q: $\mathbb{Q}[\pi^{2^n}]\subsetneqq \mathbb{Q}[\pi^{2^{n+1}}]$

Jacopo BorgaHow to show that $\mathbb{Q}[\pi^{2^n}]\subsetneqq \mathbb{Q}[\pi^{2^{n+1}}]$? I easly prove that $\mathbb{Q}[\pi^{2^n}]\subset \mathbb{Q}[\pi^{2^{n+1}}]$, now I want to prove that $\pi^{2^n}\not\in\mathbb{Q}[\pi^{2^{n+1}}]$ but I can't do that in an elegant way (my proof need a lot of calculati...

A title should not be all-MathJax; having some plain text helps with search and navigation. (autocomment)Normal Human 20 secs ago
0
Q: Is it convergent?

piternet$ \sum_{n=1}^{\infty} -\frac{1}{2^{n}} + \frac{1}{2^{n+1}+1} + \frac{1}{2^{n+1}+2} $ No idea which convergence test use for this one.

Short title. Short question. Is it convergent?
0
Q: Olympiad problem similar to Sperner's theorem, inspired by OMM 2 ( unproven conjecture of mine)

dREaMConsider a set of cubes $F$, such that each corner $(x,y)$ of any given cube of $F$ satisfies $0\leq x,y \leq n$, and each cube has a corner with coordinates $(0,0)$. What is the maximum number of cubes in $F$ so that no cube of $F$ contains another cube of $F$? So far I have managed to prove t...

0
Q: Prime Ideal and Proper Ideal

MarkCould someone please explain to me the definition of a prime ideal and a proper ideal. I honestly do not understand this concept. If possible please explain your version of the definition in the most brain dead way possible. Imagine you are trying to explain this to a little kid, how you would ...

0
Q: Compact space - definition

pipitaI have a doubt at the definition of compact spaces. So if you have a topological space X, then X is compact if every open cover of X has a finite subcover. In other words, if X is the union of a family of open sets, there is a finite subfamily whose union is X. So, if {A_i} is a family of open s...

 
12:36 AM
0
Q: Hello, I need a paper

Indalecio RuizI need a paper which explains all the process to prove that it´s enough to riffle cards 7 times. ¿Someone has a link? If you have it in spanish i will be too greatful.

Short title. Short question. Hello, I need a paper
 
1
Q: Clarify that the tag-wiki excerpt is plaintext, and don't pretend otherwise

DeduplicatorAs many visitors on meta have probably painfully learned from painful experience, tag-excerpts are plain-text. Not that the editing-help or anything else hints at that: The usage guidance, or tag wiki excerpt, is a short blurb that describes when and why a tag should be used on this site spe...

 
0
Q: Diagonalizability of a Linear Map

Arcane1729Please could somebody verify this: $T$ is diagonalizable $\iff$ there exists a basis of $V$ (the vector space ) consisting of eigenvectors of $T$ $\iff$ the algebraic multiplicity of each eigenvalue is equal to its geometric multiplicity. If even one geometric multiplicity is strictly less than ...

Question contains please. Tagged proof-verification. Diagonalizability of a Linear Map
0
Q: Consider the following vectors in R 3

user297766Consider the following vectors in R 3 v1 = 1 √ 3   1 1 −1   , v2 = 1 √ 2   1 −1 0   , v3 = 1 √ 6   1 1 2   , (a) Show that they form a basis of R 3 . (hint: compute the inner products v t i · vj ). (b) Work out the coordinates of a vector x =   x1 x2 x3   in the basis {v1 , . . . ...

Title ends with a digit. Consider the following vectors in R 3
0
Q: Integral of an operator

user297767In quantum mechanics we know that if $q$ corresponds to a complete set of parameters characterizing a quantum system, then the state vectors $|q\rangle$ satisfy the following identity: $$\int |q\rangle\langle q| d\tau_q = \Bbb 1$$ where $|q\rangle\langle q|$ is a projection operator and $\Bbb 1$ ...

0
Q: P(A|C)=P(A|B^c)*P(B^c|C), how to prove it?

whoisithis is not Bayes theorem, is it? How to prove this formula?Sorry for this naive question. I'm a first year math student.

 
0
Q: How to notify an user that can potentially answer a question?

Carlos MendozaI just found a question that I believe could receive a good answer from this user. How could I notify him or reference him in the question? I am looking for something like a comment that can get the attention of the potential answerer, not just a link to his profile.

 
0
Q: What is the sum of the expression below?

AAA$$\sum_{n=0}^{n=\infty}\frac{a^n}{(n-1)!}$$ Assuming that $a>0$

0
Q: Evaluating partial sum of $\sum_{n=1}^{\infty}{x}a^{x-1}$

melinda$$\sum_{n=1}^{\infty}{x}a^{x-1}$$ where a is between 0 and 1. can anyone explain me how it can be evaluated to be $${1}\over{1-a}$$

0
Q: How to solve the following probability question?

MATH ASKERSuppose you have five books in your book bag. Three are novels, one is biography, and one is a poetry book. Today you grab one book out without looking, and return it later. Tomorrow, you do the same thing. What is the probability that you grab a novel both days. I thought it would be $$ 3 / 5 $...

Words such as question are uninformative in titles. Please edit the title so that it better describes the specifics of your question. Do not hesitate to make it longer or include a formula if needed. More tips here. (autocomment)Normal Human 21 secs ago
0
Q: Finding all integral solutions to a particular problem

keroIm working on some problems in Arthur Engel's problem solving strategies book and one of the problems is: On page 132: the question is: Now here is what the back of the book says: So I am trying to find the other solution based on the last sentence. My guess is it would involve modular ari...

0
Q: Interesting Sum Problem

JakeCan you find some real numbers $x_1,x_2,...,x_n$ that follow $\sum_{k=1}^n{x_i^q}=q$ for all positive integers $q > 1$? Take $n$ to be some positive integer.

Words such as interesting are uninformative in titles. Please edit the title so that it better describes the specifics of your question. Do not hesitate to make it longer or include a formula if needed. More tips here. (autocomment)Normal Human 21 secs ago
 
1:25 AM
1
Q: Basis of differential one-form confusion

Peter4075I'm trying to understand one-forms in the context of general relativity. Lee (Introduction to Smooth Manifolds) says that at a point $p$ and with a vector field $X$ we define a covector field $df$, called the differential of $f$, by$$df_{p}\left(X_{p}\right)=X_{p}f.$$ My question is, is th...

Questions tend to get more attention when they have a tag for a broad area of mathematics relevant to the question. Some of these tags might fit. (autocomment)Normal Human 21 secs ago
0
Q: Is my reduced row echelon form incorrect?

tehnininessI need to find the reduced row echelon form of this matrix -8 6 -2; 6 -7 4; -2 5 -3; I performed the following row operations: r1+r2= -2 -2 2 r2+2(r3)= 2 1 -2 (-1/2)r1= 1 1 -1 r2+r3= 0 5 -5 r3+r2=0 5 -5 r3-r2= 0 0 0 (1/5)r2= 0 1 -1 r1-r2= 1 0 0 ...

This site uses MathJax formatting of formulas. More tips here. (from a bot)Normal Human 21 secs ago
0
Q: What is the correlation between X and Y?

AllieLovesMathlet $x_1, x_2, y_1$ be uncorrelated data, $$X = x_1 + px_2$$ $$Y = x_1+ qX$$ then what is the correlation between X and Y? I am guessing its q?

0
Q: Why simply connected solvable analytic groups have no nontrivial compact subgroups?

DelacroixWhy do simply connected solvable analytic groups have no nontrivial compact subgroups? I'll appreciate any help on this question.

 
0
Q: Search in Ask Question should filter or prioritize by tags

Zan LynxI just noticed a duplicate question in the Go language tag that has been repeating in various forms pretty much since forever. I thought, "Why don't people find the duplicate question when asking? Why doesn't the search help them?" I discovered that the search does not filter by tags apparently...

 
Normal Human, would you know how to put up a bot which can search for certain words each day in a given website and notify a given person?
 
1:48 AM
@AloizioMacedo This is similar to what my bot does, for example if the question body contains certain words, the question gets posted here. This is the simplest form of notification I could think of: posting in a chatroom dedicated to the purpose. The technical part: reading new posts via the API, scanning text with a regular expression. If you're not a programmer, bookmarking a search page is definitely easier.
There are also web services that offer you to track a certain page for changes. You could try that with the search results page, probably easiest.
0
Q: Generating functions an =n^2

Jonathan LeeDerive a generating function for the sequence $a_n = n^2$ I know the power series for $n^2$ is $${x(x+1)}\over{(1-x)^3}$$ However I am struggling to connect the two with the proof. Any help would be greatly appreciated thanks!

Short title. Title ends with a digit. Generating functions an =n^2
0
Q: general question about primes in $Z[\sqrt{2}]$

settheorynoobHow do I factor 2 + 3$\sqrt{2}$ into primes in $Z[\sqrt{2}]$? I know that primes are irreducible in $Z[\sqrt{2}]$ and that units are of the form $\pm(1\pm\sqrt{2})^n$. How are primes and units related, if at all? I understand that all primes of $Z[\sqrt{2}]$ are obtained by factoring rational pr...

Words such as question do not add information to titles. Please edit the title so that it better describes the specifics of your question. Do not hesitate to make it longer or include a formula if needed. More tips here. (autocomment)Normal Human 21 secs ago
0
Q: Help with finding the directional derivative of a $f(x,y,z)=xy+xz+yz$ at a point and in the direction of a vector

inquisitorI need to find the directional derivative of $f(x,y,z)=xy+xz+yz$ at $P(1,2,3)$ in the direction of $\overrightarrow{v}=\langle 2,1,-1 \rangle$ I think I started this incorrectly and would greatly appreciate any hints! $$z=\frac{-xy-xz}{y}$$ $$f_x=0$$ $$f_y=\frac{xy+xz}{y^2}$$ When I got to ...

 
2:06 AM
0
Q: why was Gauss's Lemma on number theory made and what is it used for?

smithI fully understand both the Lemma and proof of Gauss's Lemma on number theory which states $$\bigg(\frac{a}{p}\bigg) \equiv (-1)^n, $$ where $(\frac{a}{p})$ is the Legendre symbol https://proofwiki.org/wiki/Gauss%27s_Lemma_(Number_Theory) My question here is, what was the reason behind this l...

Tag (intuition) should not be the only tag a question has. Please add a tag for a subject area to which the question belongs. (from a bot)Normal Human 21 secs ago
 
2:18 AM
0
Q: finding eigenvalues and vectors for a linear transformation over a infinite dimensional Vector spaces.

user297774let V={(x_1, x_2...x_n...)}|x_i are real numbers } under normal operations and T((x_1, x_2...x_n...))=(x_1+x_2+,x_2+x_3...(x_n)+(x_n+1)...) Find T((x_1, x_2...x_n...))=lamba((x_1, x_2...x_n...)) so so i start like (x_1+x_2+,x_2+x_3...(x_n)+(x_n+1)...)=(((lamba)x_1, (lamba)x_2...(lamba)x_n......

Welcome to Math.SE, user297774. This site uses MathJax formatting of formulas. More tips here. (autocomment)Normal Human 21 secs ago
0
Q: Need help with optimization problem involving a triangle and its lengths.

Mone Skratt HenryFind the rectangle of maximum area that can be inscribed in a right triangle with legs of length a=43 and b=44 if the sides of the rectangle x,y are parallel to the legs of the triangle, as in the figure. Okay, I know that I should first draw out an image of everything so that I can visual what ...

 
2:37 AM
0
Q: Stalks and direct image

D_SLet $f: X \rightarrow Y$ be a continuous map of topological spaces, and $F$ a sheaf of rings on $X$. The direct image sheaf $f_{\ast}F$ on $Y$ is given by the formula $V \mapsto F(f^{-1}V)$. If $x \in X$, is it true in general that $F_x \cong (f_{\ast}F)_{f(x)}$? We have $$(f_{\ast}F)_{f(x)} =...

0
Q: Integer Lattice Points

britgirl5Let (n1,m1),(n2,m2),. . .,(n9,m9) be integer lattice points in the plane (ie. ni and mi are integers). Show that the midpoint of the line joining some pair of points is also an integer lattice point. I think that I need to use the pigeonhole principle but I'm not sure how to get to that point.

0
Q: Can You Give Me Help On Expected Value Please?

OverachieverThe Question: Let $X_{1}, X_{2}, ..., X_{9} $ be a random sample of size 9 from a normal distribution $N(2,4)$. Let $Y_{1}, Y_{2} , Y_{3}, Y_{4}$ be an independent random sample from a normal distribution $N(1,1)$. Let $\bar{X}$ and $\bar{Y}$ be their sample means respectively. Let's assume that ...

Words such as help, please do not add information to titles. Please edit the title so that it better describes the specifics of your question. Do not hesitate to make it longer or include a formula if needed. More tips here. (from a bot)Normal Human 20 secs ago
@AloizioMacedo If you want, my bot can ping you in this room with a link to the question, whenever it finds a keyword of interest to you.
It's a lot easier to add a line of code to an existing bot than to make a new one.
0
Q: T/F: The set: $\{(x,y): sin(x^{2012} +y^3) + x^2 + y^4 ≤ 1$ is a compact set in $R^2$.

whatarethoseT/F: The set: $$\{(x,y): sin(x^{2012} +y^3) + x^2 + y^4 ≤ 1$$ is a compact set in $R^2$. I think it is false since I don't believe the set is closed nor bounded? Is that correct?

 
3:01 AM
-3
Q: Do we have "migration vampires"?

GstestsoI'm just curious that if we have some "migration vampire" that lazy to find suitable sites (not just because they are beginner or mislooked the site logo) and hence ask anywhere to obtain migration service. Are we suffering from this problem currently?

 
0
Q: Finding the geometric series of the fraction

IdiotfromPrincetonI am confused as to how to turn a fraction into a sum using geometric series. I need to find Laurent series I have $\frac{z+2}{(z-1)(z-4)}=\frac{2}{z-4}+\frac{-1}{z-1}$ I do not know how I turn the last 2 fractions into geometric series and write them as sum. Can someone please help me?

0
Q: trying to prove Let Fn be the nth Fibonacci number. Then (Fn+1)2 - (Fn+1Fn) - (Fn)2 = (-1)n

user1787331I am trying to prove - Let Fn be the nth Fibonacci number. Then (Fn+1)2 - (Fn+1Fn) - (Fn)2 = (-1)n - I am not sure where to start with this.

 
3:17 AM
0
Q: function has no inflection point but second derivative =0

Spongebob Squareroot-pantsI got this question for a finals review: The answer says it has no inflection point, but I got the second derivative to be y''=2-2sinx And when y''=0, x=2kπ+π/2 So how can this be? Thanks. P.S. The homework tag seems to be gone for some reason, so I can't add the tag.

 
1
Q: Log in is broken

Remy LebeauI primarily use IE11. I visit StackOverflow several times a day, and just leave it logged in. Closing and reopening the browser logs me back in automatically. IE just crashed on me. When I brought it back up, I was no longer automatically logged in to StackOverflow. I guess my cookie got des...

 
3:38 AM
-1
Q: Break a large web service project into micro services

Knows Not MuchIn my company we have a spring project which has seen many years of development. It is a massive web services project with many many rest end points. The problem is that now when we are growing into multiple agile teams, working on this monolithic project is becoming painful. There are too many ...

 
0
Q: Thomson's Lamp Question

BillyKThe Thomson's Lamp paradox: A mad scientist owns a desk lamp. It begins in the toggled on position. The scientist toggles the lamp off after one minute, then on after another half-minute. After a quarter-minute the lamp is toggled off, then the scientist waits an eighth-minute and turns the lamp...

Words such as question are uninformative in titles. Please edit the title so that it better describes the specifics of your question. Do not hesitate to make it longer or include a formula if needed. More tips here. (autocomment)Normal Human 21 secs ago
0
Q: Integration over complex plane

IdiotfromPrincetonI have a problem with the following integral $$\int_{-\infty}^{\infty}\frac {x\sin x}{x^4+1}$$ Can someone please help me with the way the solution goes? I would highly appreciate it Thanks in advance!

Question contains please. Integration over complex plane
0
Q: exponential difference inequality

user3258845Im asked to prove the inequality: $0\leq a<b$ and $x>0$ $$ a^x(b-a)<{b^{x+1}-a^{x+1}\over{x+1}}<b^x(b-a) $$ So far I have seen that obviously: $$a^x(b-a)<b^x(b-a)$$ and that $$b^{x+1}-a^{x+1} = (b-a)(b^x+b^{x-1}a+...+ba^{x-1}+a^x) > a^x(b-a)$$ $$$$ I was thinking it may have to do with $a<{a+b\...

0
Q: property of supremum

turtleIt seems that the following is the basic property of the supremum of a function: $$\sup_{x \in I}f(x) - \inf_{y \in I}f(y) \geq \sup_{x, y \in I}f(x) - f(y)$$ I think This property is so obvious that the textbook just uses this without mentioning why. But I want to know if this can be proven ma...

Short title. property of supremum
0
Q: Consider P={0, 1/5, 3/10, 2/3, 3/5, 14/15, 1} Find the mesh of P.

ematth7My professor says that the answer is 1/3 but he never explained to us how he got that. Just hoping for some insight! Thank you!

0
Q: What are the unknown angles in the diagram below?

Atoosahttp://i.stack.imgur.com/a1gnS.png So I used the pythagorean theorem to find the missing leg first. a^2 + b^2 = c^2 a^2 + 6^2 = 10^2 a^2 + 36 = 100 100 - 36 = 64 √64 = 8 Now I'm just lost from here. It was just recently that I started studying tan, cos, sin etc... so any help is appreciated.

This site uses MathJax formatting of formulas. More tips here. (autocomment)Normal Human 21 secs ago
0
Q: Famous Function?

Chris ThomasI am curious if anyone recognizes the following function as belonging to a particular class of functions (like quadratics) or belonging to any particular academic laws (economic, biological, etc). It is not out of a text book or class, but I believe it could describe personal living expenses as a...

Short title. Famous Function?
 
4:19 AM
0
Q: A semicontinuous function possesses a residual set of points of continuity

James ChanIf $f:X\rightarrow\mathbb{R}$ is a semicontinuous function, then its points of discontinuity lie in the union of countably many closed nowhere dense sets. Thanks a lot.

Consider replacing (analysis) with a more specific tag for the relevant branch of analysis. (autocomment)Normal Human 21 secs ago
0
Q: Let p be an odd prime. Suppose that p divides a − b and a + b. Prove that p divides a.

user297794Need help on this proof: Let p be an odd prime. Suppose that p divides a − b and a + b. Prove that p divides a.

0
Q: Prove that if rank$\begin{pmatrix} A &B\\ C&D\end{pmatrix}$=rank(A), then $D=CA^{-1}B$ .

tinkLet A be an invertible $n \times n$ matrix with entries from a field F. Prove that if rank$\begin{pmatrix} A &B\\ C&D\end{pmatrix}$=rank(A), then $D=CA^{-1}B$ .

 
4:43 AM
0
Q: Suppose that Y is a binomial random variable based on n trials with success probability p and consider Y* = n - Y.

Richard A. TorresSuppose that Y is a binomial random variable based on n trials with success probability p and consider Y* = n - Y. a. Argue that for y* = 0, 1, ..., n P(Y* = y*) = P(n - Y = y*) = P(Y = n - y*) b. Use the result of part a to show that part b of the exercise Hi. Someone can give me a hi...

This site uses MathJax formatting of formulas. More tips here. (autocomment)Normal Human 21 secs ago
0
Q: Describe the prime factorization of all integers n such that φ(n) is a power of 2

user297794Need help in solving this: the prime factorization of all integers n such that φ(n) is a power of 2 Where φ(n) is the Euler's phi function

0
Q: Behavior of wave equation when one of the initial conditions = 0

WapitiI have gathered in my reading that there is a difference between the behavior of the wave equation depending on whether the initial position or initial velocity is zero, but my impression is not precise enough to be very clear to me. If we have the one dimensional wave equation $u_{tt} = u_{xx}...

 
5:07 AM
0
Q: Show that $d(x,y)=\dfrac{1}{k(x,y)}$ is metric .

asdasd asdLet $X$ be a non empty set. Let $M$ the set of all sequences $(x_{n})$ of elements of $X$. For $x=(x_{n})$ and $y=(y_{n})$ in $M$, let $k(x,y)$ the smallest integer $n$ such that $x_{n}\neq y_{n}$. Let $d:M\times M\to \mathbb{R}$ $d(x,y)=\dfrac{1}{k(x,y)}$ if $x\neq y$ and $d(x,x)=0$. Show that $...

Tall formulas in titles break the layout of question lists. Please replace \dfrac with \frac in the title. (autocomment)Normal Human 21 secs ago
0
Q: $f(x)=\left\{ \begin{array}{ll} x & \mbox{if $x \in \mathbb{Q}$};\\ x^2 & \mbox{if $x \in \mathbb{Q}^c$}.\end{array} \right.$

chrisw$f(x)=\left\{ \begin{array}{ll} x & \mbox{if $x \in \mathbb{Q}$};\\ x^2 & \mbox{if $x \in \mathbb{Q}^c$}.\end{array} \right.$ This question asks to find function $g$ and function $h$, which are continuous on [0,2] such that if P is a partition of [0,2] that includes the point 1, then $L(f,P)=L(g...

0
Q: Euler Characteristic

m2271rI want to show that two surfaces are not homeomorphic, say $S^2$ and $\mathbb{R}P^2$, for example. Is it enough to show that their Euler characteristis differ to conclude this?

Short title. Short question. Euler Characteristic
0
Q: Need help with understanding why $\ell^p$ is reflexive for $1 < p < \infty$?

user264885I'm having difficulties understanding the following proof: I'm used to seeing different notation (used in Erwin Kreyszig's functional analysis) where the canonical map is $C: X \to X''$ with $x \mapsto g_x$. And $g_x: X' \to \mathbb{K}$ where $f \mapsto f(x)$ In the image above having troubl...

 
5:31 AM
0
Q: Proof verification: $\mathfrak{g}$-module $V$ is irreducible if and only if $V\subseteq \mathfrak{g}(v)$ for each nonzero $v\in V$.

Irreducible-moduleI want to prove that: The $\mathfrak{g}$-module $V$ is irreducible if and only if $V\subseteq \mathfrak{g}(v)$ for each nonzero $v\in V$. Since $\mathfrak{g}(v)$ for any non-zero $v\in V$ generates a submodule of $V$, we want these to not be proper. Proof of $\iff$: $(\,\,\Longleftarrow \...

 
5:44 AM
0
Q: Calculus Cardioid word problem! Optimal pickup range of a microphone

MichellePlease help with my exam review question- thank you!! When recording live performances, sound engineers often use a microphone with a cardioid pickup pattern because it suppresses noise from the audience. Suppose the microphone is placed 2 m from the front on the stage (as in the figure) and the...

0
Q: How to obtain this partial fraction decomposition?

miltthetankI am studying Laplace transforms right now and got stuck at this step that involves a weird partial fraction decomposition. It looks like the instructor skipped a bunch of steps and assigned numerators to a bunch of the fractions without assigning them dummy variables. Any idea how he got to this...

Tagged differential-equations but mentions "partial". How to obtain this partial fraction decomposition?
0
Q: Metric Spaces Proof

RatonLet p be a prime number, and d: $\mathbb{Z} × \mathbb{Z}$ → [0, +∞) be a function defined by $d_p(x, y) = p^{−max(m∈N : p^m|x−y)}$ . Prove that $d_p$ is a metric on $\mathbb{Z}$ and that $d_p$(x, y) ≤ $max(d_p(x, z), d_p(z, y))$ for every x, y, z ∈ Z What I tried: I tried to use triangle inequal...

Short title. Metric Spaces Proof
0
Q: Method for showing a function has a branch point at infinity?

JosephLet us say I have a function $f(z)$ please can you give me the steps to show if this has a branch point at infinity and how to determine its order?

Short question. Question contains please. Method for showing a function has a branch point at infinity?
0
Q: $p$-adic logarithm, $|\log_p(1 + x)|_p = |x|_p$?

Kevin WilliamsDefine the $p$-adic logarithm$$\log_p(1 + x) = \sum_{i =1}^\infty (-1)^{i-1}x^i/i.$$How do I see that if $p > 2$ and $|x|_p < 1$, then $|\log_p(1 + x)|_p = |x|_p$?

 
6:15 AM
0
Q: Maximum reputation you can get from answering a question?

Sarath NairI know there is a maximum cap of 200 for reputation per day for a user. Is there a maximum cap that you can get for answering a question? In theory if I get a 1K votes for an answer that will get converted to 10K reputation, right? Also do I loose reputation if the post gets converted to a commun...

 
6:42 AM
0
Q: RSA Cryptography math problem

KFCI have this math problem I'm kind of stuck on. You intercept the message 27284682555982882069237 which was encrypted using a public modulus of 124137798108168664109413 and an encryption exponent 257. The modulus is now too large to be factored by testing successive candidate divisors...

Short title. Title contains problem. RSA Cryptography math problem
0
Q: Fourier Transform of a line Ax+By+C = 0

sinbagCan someone help me in a step-by-step derivation for the Fourier Transform of a line ? It appears to be simple but still cannot figure out. I know what is the end result but I am unable to figure out the intermediate steps. I tried to use the Shifting and the Similarity theorems but I am not gett...

 
6:53 AM
0
Q: a question about a set containing a nilpotent matrix

monalisaLet $S$ be the set of 3x3 matrices $A$ such that $A^tA$ = $\begin{bmatrix}1&0&0\\0&0&0\\0&0&0\end{bmatrix}$. How to prove that $A$ is of rank one and $S$ contain a nilpotent matrix? First part : since$~~$ $rank(A) = rank(A^t)= rank(AA^t) =rank(A^tA)$, first part i can solve. How to prove the...

Words such as question are uninformative in titles. Please edit the title so that it better describes the specifics of your question. Do not hesitate to make it longer or include a formula if needed. More tips here. (autocomment)Normal Human 21 secs ago
 
0
Q: Conventions for writing MathJax code

Bernard WojcikI am new to MathJax. I copied some the code from the MathJax basic tutorial and quick reference \begin{array}{ccc|c} a&0&b&2\\ a&a&4&4\\ \end{array} Then I edited it to get more rows for the matrix \begin{array}{ccc|cc} a&0&b&2\\ a&a&4&4\\ 4$2$3&2\\ \end{array...

 
0
Q: Leaky bucket problem

MaharajaXLeaky bucket problem $$\beta = 16 KB$$ $$packet\ size = 1KB$$ $$\rho = 8\ packets/sec$$ What is maximum Burst Size? Taken from here https://www.youtube.com/watch?v=4eMrQXU0DdA&list=PLpherdrLyny-zJw95jcE-uJkcsIAG1MEn&index=103 Video provides example but never solution... I did not understand h...

Short title. Title contains problem. Leaky bucket problem
0
Q: Inequality with $a,b,c\in{}\mathbb{R}$.

Jack FrostProve that for every real numbers $a,b$ and $c$ we have $$(a+b+c)^5\ge 81(a^2+b^2+c^2)abc.$$

Short title. Short question. Inequality with $a,b,c\in{}\mathbb{R}$.
0
Q: Is the first quadrant in XY plane not a smooth manifold (with boundary)?

Daum YoonI'm a beginner in smooth manifold theory and was just wondering if this was easy to prove.

0
Q: When to Stop Rolling and Bail

Sandeep SilwalConsider the following game. P has a fair $6$ sided die and continuously rolls it while keeping track of the sum, $S$, of the top faces. P can bail anytime and leave with $S$ number of dollars. However, if $S$ is ever a perfect square, the game ends and $P$ gets nothing. When is it m...

 
7:21 AM
0
Q: A simple expansion

Ali JanWhere I am doing wrong? if any one can tell!!! I have done this equation as $$y ={x\over 1+(x-x^3+x^7)^3} = x [1+(x-x^3+x^7)^3]^{-1}$$ Using binomial expansion, $$y= x[ 1-(x-x^3+x^7)^3+...]$$ $$y= x[ 1-(x^3+x^9+x^{21}-3x^5+3x^9+3x^7+3x^{13}+3x^{15}-3x^{17})+...] $$ $$y= x[ 1-x^3-x^9-x^{21}+3x^5-...

Short title. A simple expansion
0
Q: solution of first order partial differential equation

user297669Here $k$ and $k_{1}$ are constant $\frac{\partial }{\partial t}f\left(s ,t\right)=k{f}^{2}\left(s,t\right)-{k}_{1}f\left(s,t\right)$

0
Q: Evaluating area D using polar coordinates

userLet D be the region in the xy-plane bounded on the left by the line x=2 and on the right by the circle x^2 + y^2 = 16. Evaluate \iint (x^2 + y^2)^(-3/2)dA

Welcome to Math.SE, user. This site uses MathJax formatting of formulas. More tips here. (autocomment)Normal Human 21 secs ago
0
Q: $f$ and $g$ are two functions from $[0,1]$ to $[0,1]$ with $f$ strictly increasing .Then.....

user118494$f$ and $g$ are two functions from $[0,1]$ to $[0,1]$ with $f$ strictly increasing .Then which of the following is true $?$ $A)$ If $g$ is continuous then so is $f\circ g$ counterexample : $g(x)=x$ or $g(x)=1$ and $f(x)={1\over {1-x}}$ $B)$ If $f$ is continuous then so is $f\circ g$ ...

Tagged proof-verification. [$f$ and $g$ are two functions from $[0,1]$ to $[0,1]$ with $f$ strictly increasing .Then.....](math.stackexchange.com/q/1570533)
0
Q: how to find minimum/maximum of following equation?????

ajoyGiven y=(a-x)(x+√(x^2+b^2). How to see whether it is max or min and find that value also ?

Title contains ??, ??. Short question. how to find minimum/maximum of following equation?????
 
7:55 AM
0
Q: A problem in vectors with a pyramid whose base is a parallelogram

Yaron Cohen-Taldrawing In the drawing, $ABCDS$ is a pyramid whose base is a parallelogram. $O$ is the intersection of the parallelogram diagonals. The following holds: $\overrightarrow{SF} = k \cdot \overrightarrow{SD}$ $\overrightarrow{SE} = t \cdot \overrightarrow{SO}$ The problem is to express $t$ using...

0
Q: How can I prove that $R$ is complete using the construction with Cauchy sequences

mattHow can I prove that $R$ is complete using the construction with Cauchy sequences ?

0
Q: Disprove why 0 ∉ Z

Ajeet KljhA value x is said to be an integer when floor(x) = x, where x ∈ ℝ or more equivalently floor(x)/x = 1 Since 0 ∈ ℝ, and we assume that 0 is either an integer or non-integer, from the same equation to determine whether or not 0 is an integer we find: floor(0) = 0 Since floor(0) takes the highes...

Short title. Disprove why 0 ∉ Z
0
Q: MATLAB plotting log(h) vs log(e(h))

Stan-LeeSo I have the initial value problems below. From a previous part, I have already confirmed that v(t) is a solution to the the problem above it. Now I am left to write a MATLAB code that plots log(h) vs log(e(h)) and that verifies that the Euler Method's approximation is first order by computing t...

 
0
Q: Undo a "Reject" on a suggested edit

Mr ListerI saw that a question had a pending edit, but the edit made no improvement whatsoever. So I clicked "Reject", at the same time I realised that the question did actually need some editing. I should have clicked "Reject and Edit"! But by now there's nothing I can do to edit the question, other th...

 
8:24 AM
0
Q: Proof the convergence of a random variable

useprxfGiven $x_n \sim N(0, \frac{1}{n})$, does $x_n$ almost sure converge or converge in probability or converge in distribution? How to prove it

0
Q: On degenerate linear systems

TurboIs it true that if in $$AX=0$ where $A$ is $n\times n$ matrix over field $\Bbb K$ and $X$ is a length $n$ vector of variables if $rank(A)=n-1$ we will have an unique solution?

Short title. Short question. On degenerate linear systems
0
Q: Evaluate $\lim\limits_{x\to\infty} \dfrac{[x]+x}{x}$

WarriorEvaluate $\lim\limits_{x\to\infty} \dfrac{[x]+x}{x}$, where $[x]$ denotes greatest integer $\leq x$. We have, if $[x]=k$, then $k\leq x<k+1$. But I don,t know how to evaluate this. Any hints?

Tall formulas in titles break the layout of question lists. Please replace \dfrac with \frac in the title. (autocomment)Normal Human 21 secs ago
 
8:39 AM
-3
Q: Asking Question for the first time made me wait for 90 minutes

NickalIts acceptable that one need to have a gap of 90 minutes between questions. But when I was posting my very first question, I was asked to wait for 90 minutes. Why is it so? Or, is it an error?

 
0
Q: Probability of rank $r$ matrix

TurboSample $n^2$ integers $a_{11},\dots,a_{nn}$ in $[-d,d]$ uniformly. What is the probability that the resulting matrix $[a_{ij}]$ has rank $r$?

 
0
Q: Is there point in re-tagging old questions after tag creation?

TimI just created implementation since it seems that many questions could possibly fall into this category and it would make it easier to find similar questions. I started wondering, would it make sens to re-tag some old questions with this tag now? This would bring many old questions on the top of ...

 
0
Q: Example for geometric realization on semi-simplicial set that doesn't preserve limit

Bipolar MindsI'm looking for a diagram $D$ (as simple as possible) in the category of semi-simplicial sets (i.e $sSet$ with only monos) such that $R(\text{lim}\,D) \ncong \text{lim}\,R(D)$, where $R$ is the geometric realization on semi-simplicial sets. Moreover, in $sSet$ the equivalence should be true, i.e....

Consider adding a tag for a broader subject area to which the question belongs. Some of these tags might fit. (autocomment)Normal Human 20 secs ago
0
Q: Is this a valid proof that the sum of the angles in a triangle is $180^\circ$?

Александър ГьоревI think I accidentally found a proof of the famous theorem that the sum of the angles of a triangle add up to $180 ^\circ$, but am not sure if it is correct. Here it is: It can be proved that the median in a right triangle is equal to half the hypotenuse. Consider an aritrary triangle $QRS$ , $...

 
9:27 AM
0
Q: Integration of |e^-(2+j)t |^2

Dushani WellappiliThe integration of |e^-(2+j)t |^2 from zero to infinity is 1/4 when I seperate above as |e^-2t|^2 * |e^-2jt|^2 and integrate. |e^-2jt| was taken as 1. But when I integrate the problem taking |e^-(2+j)t|^2 = e^-(4+2j)t the answer for the integral is 2-j /10. Can some one tell what is the correct...

Welcome to Math.SE, Dushani Wellappili. This site uses MathJax formatting of formulas. More tips here. (autocomment)Normal Human 21 secs ago
0
Q: Graph Theory- show maximum number of edges in a simple graph

Salim DarjaanShow that the maximum number of edges in a simple graph with n vertices is {n(n-1)\over 2} ?

 
0
Q: Are late comments requesting jsFiddle links in answers really okay?

Frédéric HamidiI recently received a comment under an older (2012), jQuery UI-related answer of mine. The comment in question only consisted in: please share a working fiddle I had a hard time understanding the rationale behind that request, given that: The question itself did not contain either HTML ma...

 
0
Q: Find if they are inverse matrices are not ?

samanthaI'm new to this site, Can someone explain if and if not Image to the question

 
9:45 AM
0
Q: Bug in Reputation Graph

N-JOYIssue: Incorrect mapping between reputation graph and posts from which user earned the corresponding reputation Detail: Bars shown in Reputation graph have incorrect mapping with Post from which reputation has been earned. for more details look at these attached images: Here reputation earn...

 
10:00 AM
0
Q: Let $S$ be the set of all real numbers $r$ such that $f(g(x))=g(f(x))$ for infinitely many real number $x$.Find the number of elements in set $S.$

Vinod Kumar PuniaLet $f(x)=\frac{x}{1+x}$ and let $g(x)=\frac{rx}{1-x}$ Let $S$ be the set of all real numbers $r$ such that $f(g(x))=g(f(x))$ for infinitely many real number $x$.Find the number of elements in set $S.$ I found out $f(g(x))=\frac{rx}{1-x+rx}$ and $g(f(x))=rx$ But i do not know how to solve it f...

0
Q: lebesgue integral in D= circle

maxandriI have this doubt that I cannot solve. ∫|x−1|^a/|x2−y2|^b dxdy in D= circle x^2+y^2<1 If I use polar coordinates I cannot solve anything. Could you help me? Thank you

Welcome to Math.SE, maxandri. This site uses MathJax formatting of formulas. More tips here. (from a bot)Normal Human 21 secs ago
0
Q: Why $\mathbb{P}(X>n)=q^n$ in geomtric distribution

NehoraiLet $X\sim G(p)$ Why $\mathbb{P}(X>n)=q^n$ in geomtric distribution, I know that $\mathbb{P}(X=n)=(1-p)^{n-1}p$

0
Q: Rearrange letters in "ENGINE" so no letter appears next to itself

Josue EspinosaHow many ways are there to arrange the letters in "engine" so that no letter appears next to itself? Initially, I know that there are 6! possible arrangements of the letters. But we have to divide the 6! by 4 because there are 4 duplicate letters. There are 2 e's and 2 n's, so I must subtract th...

This site uses MathJax formatting of formulas. More tips here. (autocomment)Normal Human 21 secs ago
0
Q: Finding the reccurence relation from a problem.

Yves Halimilet $f(n)$ be the number if ways to lay down tiles in a formation of size 2 x n using tiles of size: $$ \begin{matrix} 1 \\ 1 \\ \end{matrix} $$ and tiles of size: $$ \begin{matrix} 1 & 1 & 1 \\ 1 & \\ \end{matrix} $$ Turning and ...

0
Q: Find vectors x and y with given norms.

CandiceI've spent many hours on this and I just can't understand how to do this. Could you please go through this with me? I have a test, and I really need to understand how to do these types of problems. Thank you so much!! "Find vectors x and y with sum-norm of x = 1, max-norm of y = 1, such that 1-n...

Question contains please. Find vectors x and y with given norms.
 
3
Q: Is it wrong to append a comment to question?

IvarIn this question the OP posted a question, without any error messages. After asking about the error messages he provided it, in the comments. Since this error message can be useful in order to answer the question, I suggested an edit, moving his error message to the question. To my surprise, my ...

0
Q: Must an answer be a copy paste answer?

swidmannI thought NO. Just today I flagged a question as duplicate. The OP said, this does not solve his problem, because of a reason I can not comprehend. I went to my flag history and saw that this flag is disputed. The relevant part of the answer, which I thought it was a duplicate from is: Rewrite...

 
10:24 AM
0
Q: Spherical Triangle — Angle from one Point without using North

DominikI'm designing a tool for students and right now im working with coordinates. I have the plan to shift the north pole in my code to Munich and I would like to do this by working with a spherical triangle and the angle at one point. As result I would like to have a formula which tells me how to cal...

Welcome to Math.SE, Dominik. Consider adding a tag for a broader subject area to which the question belongs. Some of these tags might fit. (from a bot)Normal Human 21 secs ago
 
10:49 AM
0
Q: Infinite series question- Diverges but does it converge conditionally? (Root test)

LidorAHello I had this question below on a midterm on Calculus I had proved the series diverge by testing the Absolute value of the series. By using the Root test which found the limit to be: e^2/3> 1 Is it necessary to test if it converge conditionally? The question This is the the prove by symb...

0
Q: Group actions - modulo 4

LindaI am having a bit trouble understanding group actions. if I am given a set A = {a,b,c,d} and a group action s: Z mod 4 -> $S_A$, how would one then be able to show if there exists a group action s such that s(2) = (a b). The only thing I seem to be able to get out of this information is that: $s...

Short title. Title ends with a digit. Group actions - modulo 4
 
11:06 AM
0
Q: A is positive definite if and only if all the principal minors of A are positive.

nupur nandiniLet A be a symmetric n x n matrix. Then A is positive definite if and only if all the principal minors of A are positive.

Consider adding a tag for a broader subject area to which the question belongs. Some of these tags might fit. (autocomment)Normal Human 21 secs ago
0
Q: Principal ideal question

XPenguenJust started learning a little bit about group theory. Can anyone give me a hint (or counterexample if the statement isn´t true). Let be $P $ Principal ideal then every Ideal $P^{'}\subset P$ is also a Principal ideal. May be a stupid question to ask but I need to know.

Words such as question are uninformative in titles. Please edit the title so that it better describes the specifics of your question. Do not hesitate to make it longer or include a formula if needed. More tips here. (from a bot)Normal Human 21 secs ago
 
0
Q: Inquisitive Badge - Downvote and then Updownvote

FabioI wonder how the Inquisitive badge process works in case of a downvote which eventually has been "updownvoted". Did the positive question record break when the downvote occurred?

 
11:26 AM
0
Q: Find the number that fits into this relation

guest83Which number fits into this geometrical symbolic relationship? enter image description here

0
Q: V is isomorphic to U . U is banach iff V is banach

NargesV is isomorphic to U as vector space. U is banach if and only if V is banach.

0
Q: Closed form for $\sum_{n=1}^\infty \frac 1 {2^n - 1}$

SekotsIs there a closed form for this?$$\sum_{n=1}^\infty \frac 1 {2^n - 1}$$ Tried searching but couldn't find anything.

0
Q: Conditional Probability and Independence

An old man in the sea.Let $A,B,C$ be events, and $B,C$ be pairwise independent. Can I say that $P(A|B,C)=P(A|B)$? The only thing I get is that $P(A|B,C)=P(A|B)P(A|C)$ Any help would be appreciated.

 
2
Q: Room Owners can star(real star, not pin) their own messages

Kevin GuanIf we pin a message two times, it'll become a star instead of pin. For exmpale: I posted a message: I'm about to pin it: It has been pinned now: But if I click pin this message again: The pin'll be removed, but there's a star on it:

 
11:42 AM
0
Q: If A is a 3x2 matrix and B is a 2x3 matrix why is AB not invertible

DanniI am getting nowhere with this question. I found a similar question on this site but I need help in a more step by step way. I would be very grateful.

Short question. Question contains step by step. If A is a 3x2 matrix and B is a 2x3 matrix why is AB not invertible
0
Q: Proving Euler's formula using infinity sums

Ole PetersenI want to prove $e^{i \cdot \pi \cdot x} = \cos x + i \sin x$. Proof: $e^{i x \pi} = \sum \frac{x^{n} i^{n} \pi ^{n} }{n!} = -i\sum_{odd} (-1)^{n} \frac{x^{2n+1} \pi ^{n} }{(2n+1)!} + \sum_{even} (-1)^{n} \frac{x^{(2n)!} \pi ^{n} }{(2n)!} $ and this should give the taylor serie for $\sin$ an...

0
Q: A bounded function having I.V.P. but not Riemann Integrable.

neelaI am searching an example of a function $f$ on $[a,b]$ such that $f$ is a bounded function having intermediate value property but is not Riemann Integrable on $[a,b].$ Please give me such type simplest example which can easily be visualized. Thanks in advance.

0
Q: Inequality for the entropy and every tree

BahbiShow that for every ε>0 there is a probability distribution p so that for every tree B: L(B,p)>H(p)+1−ε.

 
12:12 PM
0
Q: why S in SVD is a vector instead of a matrix?

Marco DinatsoliI know that when applying SVD on a matrix (m * n) I should have these three outputs: S: m × n diagonal matrix with non negative numbers U: m × m orthogonal matrix V: n × n orthogonal matrix but when using R statistical package. I got for S a vector instead of a matrix: look please: this is t...

Consider adding a tag for a broader subject area to which the question belongs. Some of these tags might fit. (autocomment)Normal Human 21 secs ago
0
Q: dirichlet expectation logarithm

m b how I can compute expectation of Dirichlet distribution of... \operatorname{E}[\ln X_i] = \psi(\alpha_i)-\psi(\textstyle\sum_k \alpha_k) can someone explain it to me in detail!? https://en.wikipedia.org/wiki/Dirichlet_distribution

Short title. Short question. dirichlet expectation logarithm
0
Q: Bcount on Maple

Johnny EvansI need to complete the definition of bcount so that bcount(n) returns the total number of odd coefficients n k , 0 ≤ k ≤ n. For instance, the values of n k for n = 6, with odd values highlighted, are: 1, 6, 15, 20, 15, 6, 1, bcount:= proc(n::TYPE) description "Count odd binomial coeffic...

Welcome to Math.SE, Johnny Evans. Consider adding a tag for a broader subject area to which the question belongs. Some of these tags might fit. (autocomment)Normal Human 20 secs ago
 
0
Q: Average Question Views

JimCouldn't find this asked already, can a query be made for viewing the average view count for questions in a given time frame? For example, x number of views on average within 1 year, y number for 2 years, et cetera. It should logically be restricted to a per tag category for more a more localize...

 
12:28 PM
0
Q: WHAT IS Inverse Laplace Transform of EXP(s^1/2)

Abdulrhman GhubbarI FOUND THE SOLUTION IN THE WOLFRAM I NEED STEPS SOLUTION

0
Q: Characterization of homogeneous polynomials

HaroldFI'm studying on Lang's Algebra the formal definition of polynomial's ring, in particular I'm at the chapter of several variables polynomials. Let $A$ be a commutative ring with unity ($1$), so we have associated at $A$ the ring $A[X_1,X_2,...,X_n]$ Define $X:=(X_1,X_2,...,X_n)$ and $f(X) \in A[X...

0
Q: Lyapunov Equation

codeNewbieI have a Matlab code which solves the Lyapunov equation $AX + XA^T + Q =0$ for a 3-D array of Matrices $A$ using matlab function 'lyap(A,Q)' . My problem is, sometimes the resultant Matrix $X$ is positive definite and sometimes it is not. My question is -- given that the matrix $Q$ is positive de...

Short title. Lyapunov Equation
0
Q: separation of variables to derive the solution $u(x, t)$

ASD123How do I solve this question, what are the steps ? Use the method of separation of variables to derive the solution $u(x, t)$ to the equation for a vibrating string $\frac{\partial ^2u}{\partial \:t^2}=9\:\frac{\partial ^2u}{\partial x^2} , (0 < x < 4, t > 0),$ with fixed endpoints $u(0, t) = ...

Tagged differential-equations but mentions "partial". separation of variables to derive the solution $u(x, t)$
0
Q: How to integrate $\displaystyle\int\sqrt{q_m}dG_m $

ketum How to integrate $\displaystyle\int\sqrt{q_m}dG_m $ if $F_n(0)=1-a_n, G_n(0)=1-b_n$ and $q_n=\frac{dF_n}{dG_n}$ It is written that $F_n, G_n$ are ''concentrated'' on $\{0,1\}$ the result should be $\sqrt{(1-a_m)(1-b_m)}+\sqrt{a_m+b_m}$, does it make sense to have an expression like $\int\sqr...

Tall formulas in titles break the layout of question lists. Please remove \displaystyle in the title. (from a bot)Normal Human 20 secs ago
0
Q: why ∫√((sin x)^2).dx = ∫|sin x|.dx

defcon2May be this is a stupid question but why ∫√((sin x)^2).dx = ∫|sin x|.dx instead of +-∫sin x.dx ? I think may be because it violates the rule that a function can't have more than 1 output for a single input, which brings me to my next question does the intgrand need to be a function?

This site uses MathJax formatting of formulas. More tips here. (autocomment)Normal Human 20 secs ago
0
Q: Show the equality $\frac{d\mu_n}{d\nu_n}=\prod\limits_{m=1}^{n}q_m$

derivative Show the equality $\frac{d\mu_n}{d\nu_n}=\prod\limits_{m=1}^{n}q_m$ I don't understand why $\frac{d\mu_n}{d\nu_n}=\prod\limits_{m=1}^{n}q_m$ with the definitions; $F_n=\mu(\xi_n\le x)$, $G_n=\nu(\xi_n\le x)$ and $q_n=\frac{dF_n}{dG_n}$, $\mathcal F_n=\sigma(\xi_m:m\le n)$ and $\mu_n,\nu_n$...

0
Q: Integration on a moving surface

SashaI consider the surface $N^{\epsilon}_t = \text{graph}\{ u/\epsilon - t/\epsilon\}$ for all $t >0$. Let $\nu$ be the outward unit normal to $N^{\epsilon}_t$ and $H$ the mean curvature of the surface. I also introduce a cutoff function $\phi \in C^2(\mathbb{R})$ such that $\int \phi =1$. I have ...

Tagged proof-verification. Integration on a moving surface
0
Q: Uncertainity regarding the proof from Rosenlicht's "Introduction to Analysis"

luka5zI would be very grateful if somebody could explain me the part in the red rectangle in the proof below. I don't quite get it why the author claims that if $\delta$ is sufficiently small then $p+1\le q-1$. I don't get it why it is the consequence of the inequality $\beta-\alpha \le x_q-x_{p-1}<(q-...

0
Q: latex template of Geometry and Topology journal

1234I need .tex file of a template paper of Geometry and Topology http://msp.org/gt/about/journal/submissions.html, can someone send me Thanks

0
Q: Mathematical Induction question Proof divisble by 4

TheUltimate AliUse mathematical induction to prove that 5^n + 9^n + 2 is divisible by 4, for n is a positive integer.

Words such as question do not add information to titles. Please edit the title so that it better describes the specifics of your question. Do not hesitate to make it longer or include a formula if needed. More tips here. (autocomment)Normal Human 21 secs ago
0
Q: Pi in imaginary numbers?

10 RepliesLook at the result of (-1)^(1/10000000)on the google calculator You should get 1 + 3.14159265 × 10-7 i Why does Pi occur in imaginary number operations that don't include Pi?

Short title. Short question. Pi in imaginary numbers?
0
Q: How do you distribute this negative?

JohnSo I have -(x - 2)² Do I rewrite it as -(x - 2)*-(x - 2) and distribute the negative to the inside making it (-x + 2)(-x + 2) or add the negative at the end of doing FOIL?

0
Q: Convexity of variance

RezlenHow can I prove the following function is convex? $\psi (x)=(x_1-(\frac{x_1+x_2+x_3}{3})^2+ (x_2-(\frac{x_1+x_2+x_3}{3})^2+ (x_3-(\frac{x_1+x_2+x_3}{3})^2$

Short title. Short question. Convexity of variance
0
Q: When t goes to infinity in function -2 * (t*e^-t + e^-t )

Dushani WellappiliWhat the answer when t goes to infinity for this function -2 * (t*e^-t + e^-t )

0
Q: If $f \in C^{\infty}$ and $f^{(k)}(0)=0$ for all integers $k \ge 0$, then $f \equiv 0$.

Vikrant DesaiI thought this was true since, $f(x)=f(0)+f'(0)x+f''(0) \frac {x^2}{2!} + \dots$ But I am wrong. Where did I make mistake?

0
Q: Pseudometric from nested sets

GodotAssume that $X$ is a nonempty set and that for every $x\in X$ we have a nested family of nonempty sets: $\{E_{x,t}: t\in [0,\infty)\}$ for which $t_1\leq t_2$ implies $E_{x,t_1}\subseteq E_{x,t_2}$, $E_{x,t}\neq\emptyset$ for all $t$. Nothing more is assumed. Is there always a pseudometric $d$ o...

0
Q: A function $f(x)$ satisfies the condition,$f(x)=f'(x)+f''(x)+f'''(x)+f''''(x)+......\infty$,where $f(x)$ is a differentiable function indefinitely

Vinod Kumar PuniaA function $f(x)$ satisfies the condition,$f(x)=f'(x)+f''(x)+f'''(x)+f''''(x)+......\infty$,where $f(x)$ is a differentiable function indefinitely and dash denotes the order of the derivative.If $f(0)=1$,then find the function $f(x).$ $f(x)=f'(x)+f''(x)+f'''(x)+f''''(x)+......\infty$ $f'(x)=f'...

0
Q: Supremum and infimum - Are those examples right?

hprandomHello did a few exercises about supermum and infimum but im not sure if my solutions are correct. The following set are given: (i) {n is element of the whole numbers | n³ > 10} (ii) {n is element of the whole numbers | (3/n) +4} (iii) {x element of real numbers | x² < 5} (iv) {k element of in...

Welcome to Math.SE, hprandom. This site uses MathJax formatting of formulas. Questions tend to get more attention when they have a tag for a broad area of mathematics relevant to the question. Some of these tags might fit. More tips here. (from a bot)Normal Human 21 secs ago
0
Q: Does $\Bbb E[X|Z]=\Bbb E[Y|Z]$ if $X,Y$ are identically distributed random variable?

TonyDoes $\Bbb E[X|Z]=\Bbb E[Y|Z]$ if $X,Y$ are identically distributed random variable, where $Z$ is a third random variable? Thank you!

Short question. [Does $\Bbb E[X|Z]=\Bbb E[Y|Z]$ if $X,Y$ are identically distributed random variable?](math.stackexchange.com/q/1570836)
0
Q: Delta function in the sense of distributions

DariusI have a problem understanding the meaning of the delta-function on the sense of distributions. E.g. I have the following equation: $(\frac{d}{dt} \theta(t) ) f(t) = \delta(t) f(t)$. What does this mean for the function f(t)? I cannot really understand what the delta function does here in t...

Consider adding a tag for a broader subject area to which the question belongs. Some of these tags might fit. (from a bot)Normal Human 21 secs ago
0
Q: Knowing $2xf(x)+x^2f'(x) = \frac{e^x}{x}$ and $f(2) = \frac{e^2}{8}$. Does f(x) has a local maximum or local minimum?

jiegec$2xf(x)+x^2f'(x) = \frac{e^x}{x}$. $f(2) = \frac{e^2}{8}$. And how to prove it? What I can get is $f'(2) = 0$ but that's not enough.

0
Q: Exponential series approximation

karakuscI have a series of the following form: \begin{align} \sum_{k=2}^\infty \left( 1 - e^{-ns^{k-1}} \right)^k \end{align} where $0<s<1$. I would like to compute an approximation of this series, for the regime where $n$ is large. One natural thing to try would be to use the approximation $1-e^{-x} \...

 
1:53 PM
1
Q: How do you tag a question about a programming language which has synonyms?

Michele Giuseppe FaddaSometimes a very popular programming language shares the same name of a more "obscure", specialized one. You would like to ask a question about the least popular one. E.g.: Swift is a mobile programming language designed by Apple, recently open-sourced. Swift (Parallel Scripting Language) is a...

 
2:06 PM
-2
Q: Jon Skeet breaks user card badge count in a different way

ryanyuyuJon Skeet previously broke badges by having the count run off his profile card. But with some new styling, it text wraps to the next line, which enlarge both his card and any others on the same line: While I can now read how many badges he actually has, the resizing of the user card is we...

 
0
Q: curl free vector field on non-simply connected domain

user297865I'm looking for an example of a curl free vector field on a non-simply connected region which is still a gradient.

Welcome to Math.SE, user297865. Questions tend to get more attention when they have a tag for a broad area of mathematics relevant to the question. Some of these tags might fit. (from a bot)Normal Human 21 secs ago
0
Q: How to determine e^-t *(cost +jsint) is periodic

Dushani WellappiliX(t) =e^-t * (cost+jsint) determine x(t) is periodic or nonperiodic and the period of its periodic

 
0
Q: Does the question counter update in real time?

Zoltán SchmidtI was just looking around the Meta, when I found this amazing page of the website. It is damn fascinating, though I'm interested in something. Does it count questions, solutions and all the stuff real-time? No need to go into details, if you are not desired, because I'm sure there's an efficient...

0
Q: Review Audit Failure

BurgiI just failed a review audit on this Hard drive clicking for 16 times during startup I was just adding a comment to ask the OP if they could add their screenshots to the SE Imgur. Other than that I didn't see a problem with the question. The review audit does not explain what I failed on. I can...

 
2:22 PM
0
Q: Gale-Ryser reformulation for a match problem

MarkI've got this problem: You are given a grid n X n with n rows and n columns. For every row i you are given number r_i and for every column j you are given number c_j . ...

Welcome to Math.SE, Mark. Tag (proof-verification) should not be the only tag a question has. Please add a tag for a subject area to which the question belongs. (from a bot)Normal Human 21 secs ago
0
Q: Why is $\Pr(Z_n=k, n\ge N)=0$

ketum Why is $\Pr(Z_n=k, n\ge N)=0$ if $\Pr(\xi_i^m=1)<1$ and $\mu=1$ for any $k>0$ It is a theorem from Durrett PTE $Z_n$ is a Galton-Watson process with $Z_{n+1}=\xi_1^n+\dots+\xi_{Z_n}^n$ with $\xi_i^m$ iid nonnegative integer valued. In the proof it is stated that: $Z_n$ is a martingale conve...

0
Q: Prove $N\subset H$.

Sucre Question: Let $H,K,N$ be subgroups of a group $G$ such that $H\leq K$, $H\cap N=K\cap N$ and $HN=KN$. Show that $H=K$. Here is my attempt: $H=K\Leftrightarrow$ $H\subseteq K$ and $K\subseteq H$. Clearly, $H\subseteq K$ as $H\leq K$. To show $K\subseteq H$; $HN=KN\Rightarrow k=hn$ where $k...

Short title. Prove $N\subset H$.
0
Q: Checking which Algorithm is used by GAP for its working

gxydHow can i check which Algorithm is used by GAP for its working. Like ` gap> p_1:=(1, 2, 4, 5);; gap> p_2:=(1, 2, 3);; gap> g_1:=Group(p_1, p_2);; gap> p_3:=(1, 2, 4, 3);; gap> p_4:=(1, 2, 5, 4, 3);; gap> g_2:=Group(p_3, p_4);; gap> g_1 = g_2 True Which Alg...

This site uses MathJax formatting of formulas. More tips here. (from a bot)Normal Human 21 secs ago
 
2:49 PM
0
Q: Find all ordered triples

Puzzled417Problem Suppose $351_7=aca_b$ for positives $a,b,$ such that $b-1=c$. Compute the ordered triple $(a,b,c).$ Here is what I tried: $351_7 = 183 = a*b^2+c*b+a = a(b^2+1)+c*b = a(b^2+1)+(b-1)*b = b^2*a+b^2-b+a \implies b^2*a+b^2-b+a -183 = 0 \implies 1-4(a)(a-183) \geq 0.$ I am not sure if th...

0
Q: An interesting geometrical relationship question

adrien88What symbolizes the number and what is the number missing from the relation?enter image description here

Words such as interesting, question do not add information to titles. Please edit the title so that it better describes the specifics of your question. Do not hesitate to make it longer or include a formula if needed. More tips here. (from a bot)Normal Human 21 secs ago
 
2
Q: Offer a bounty and add details to a question

Ozair KafrayI have been stuck in a problem for more than 2 days. There is already a similar question, in which the user though intended to know the answer a little bit differently, but that is only evident from comments to Q & A. There isn't an accepted answer, though the question is about 4 years old. I d...

 
0
Q: Limit of Equation

NoamProve that the $\lim_{n \rightarrow \infty} \frac{2^{n} n!}{n^{n}} = 0$ The Eqaution is equal to $\frac{2^{n} n!}{n^{n}} = $ $(\frac{2}{n})^{n} n!$ Its possible to say that $\lim_{n \rightarrow \infty} $$\frac{2}{n}$ is $0$ and because of this reason $\lim_{n \rightarrow \infty} $$(\frac{2}{n})...

Short title. Limit of Equation
0
Q: Lemma: x.(Ay) = (Ax).y

AAS.NAs simple as this may sound, I just do not understand what this statement implies. An nxn matrix A is symmetric if and only if: $$\bar{x}.(A\bar{y}) = (A\bar{x}).\bar{y}$$ Why is this true, and what does it even signify?

0
Q: Carnality of an ordered pair

MatthewFind the cardinality of the set of all points in the plane which have one rational and one irrational coordinate. Justify you answer My thoughts so far. We know that $Q$, the set of all rational numbers has a cardinality of $aleph_0$. Also, since the set of irrational numbers is just the reals ...

Short title. Tagged proof-verification. Carnality of an ordered pair
0
Q: Solving coupled PDEs numerically

mk112358I have the following system of PDEs for which I have given parameters $\gamma, \tau$ and $\mu$, $$\begin{align} T_t = &\ \gamma\,(L +\tau F-T)\\ F_t = & -F_x-(F-LT)\\ L_t = &\ \mu L_{xx}+(F-LT)\end{align}$$ with no flux boundary conditions at $x=0$ and $x=\infty$ for $F$ and $L$. The initial c...

Tagged pde, differential-equations. Solving coupled PDEs numerically
0
Q: Negation of a set

Joshua MathewsThe question is: Let S be a set, R be a binary relation on S, and x an element of S. Express in English the negation of the statement “For all x in S, xRx”. I was originally thinking since the negation is just the opposite, I would switch S and R to get an expression of "For all x in R, xSx"

Short title. Negation of a set
0
Q: Help me to prepare for mathamatical olympiad

AyushakjI want to appear in$Regional Mathematical Olympiad$Help me to prepare for it successfully so that I can clear that exam in one attempt

Words such as help do not add information to titles. Please edit the title so that it better describes the specifics of your question. Do not hesitate to make it longer or include a formula if needed. More tips here. (autocomment)Normal Human 20 secs ago
0
Q: Stuck with project euler 12

user297875i'm tackling project euler problems to learn to code, and this is the problem i'm trying to solve now (number 12): The sequence of triangle numbers is generated by adding the natural numbers. So the 7th triangle number would be 1 + 2 + 3 + 4 + 5 + 6 + 7 = 28. The first ten terms would be: 1, ...

Welcome to Math.SE, user297875. Words such as stuck are uninformative in titles. Please edit the title so that it better describes the specifics of your question. Do not hesitate to make it longer or include a formula if needed. More tips here. (from a bot)Normal Human 20 secs ago
 
3:27 PM
0
Q: Research Assistant badge progress incorrect?

user3071284In my profile, it says I have 8/50 tag wiki edits towards the badge. In this query, it says I have 16. Yesterday, I had 8 tag wiki edits, made 3 more, but don't see them added in to total 11 yet. What could the issue be?

 
3:39 PM
0
Q: d'Alembert functioal equation: $f(x+y)+f(x-y)=2f(x)f(y)$

Mohsen ShahriariThe d'Alembert functioal equation is: $$f(x+y)+f(x-y)=2f(x)f(y)\tag0$$ This equation plays a central role in determining the sum of two vectors in Euclidean and non-Euclidean geometries. Is there a good characterization of the solutions of this equation?

Consider adding a tag for a broader subject area to which the question belongs. Some of these tags might fit. (from a bot)Normal Human 21 secs ago
0
Q: Short prove that sequense sin(n) diverge

Boris ModelWhat two convergent subsequences immediately prove that sequense sin(n) diverge?

0
Q: Are math definitions iff statements?

johnny09I was wondering if definitions in mathematics are "if and only" statements? I know for sure that theorems are not "iff" statements. Thank you in advance for your help.

Tag (terminology) should not be the only tag a question has. Please add a tag for a subject area to which the question belongs. (from a bot)Normal Human 21 secs ago
0
Q: Does "Doing a thing to both sides of an equation" have a name?

ColinKA two part question. 1 True or False: when working with an equation or inequality, everything that you do is either: a substitution, or an operation performed on each side Note that algebraic or numerical simplifications are substitutions - $2+2=4$, so we're free to substitute $4$ where $2+...

Tag (terminology) should not be the only tag a question has. Please add a tag for a subject area to which the question belongs. (autocomment)Normal Human 21 secs ago
0
Q: Algebraic K-theory study.

Angel BlascoWich are the "best" books to initiate in algebraic K-theory study? I've got Algebraic K-theory by Bass,H and K-theory by Atiyah. Websites will be apreciated too. Thanks.

Short title. Short question. Algebraic K-theory study.
0
Q: Positive reach..

M.B.Let $M$ be a smooth closed $d$-manifold embedded in $\mathbb{R}^n$, and let $d_M: \mathbb{R}^n\to \mathbb{R}$ be given by $d_M(x) = \min_{m\in M} d(x,m)$. Define $M\oplus \epsilon = \{ x\in \mathbb{R}^n \mid d_M(x) \leq \epsilon\}$. The reach of $M$, ${\rm Reach}(M)$, is the largest $\epsilon$ ...

Short title. Positive reach..
 
00:00 - 16:0016:00 - 00:00

« first day (39 days earlier)      last day (535 days later) »